- PowerScore Staff
- Posts: 5972
- Joined: Mar 25, 2011
- Fri Sep 16, 2016 9:14 am
#90419
Complete Question Explanation
(The complete setup for this game can be found here: lsat/viewtopic.php?t=980)
The correct answer choice is (C)
If G does not sit in row 1, from the first rule G must sit in row 2 and H must sit in row 3. I must then occupy a window seat in row 2 or 3:
This information eliminates answer choices (A) and (D).
The only remaining aisle seat is in row 1, and if M were to occupy that seat, then K would have a window seat, which would lead to a violation of the fourth rule. Thus, M cannot sit in the row 1 aisle seat, which eliminates answer choice (E).
The remaining two answer choices—answer choices (B) and (C)—address row 2, of which the window seat is the only remaining seat available. If K sits in the row 2 window seat, then from the fourth rule M would have to sit in the row 3 window seat, forcing I to sit in row 1. But, this causes a violation of the condition in the question stem, and thus K cannot sit in row 2. This information eliminates answer choice (B), and thus answer choice (C) is correct.
The following hypothetical shows that answer choice (C) is possible:
(The complete setup for this game can be found here: lsat/viewtopic.php?t=980)
The correct answer choice is (C)
If G does not sit in row 1, from the first rule G must sit in row 2 and H must sit in row 3. I must then occupy a window seat in row 2 or 3:
This information eliminates answer choices (A) and (D).
The only remaining aisle seat is in row 1, and if M were to occupy that seat, then K would have a window seat, which would lead to a violation of the fourth rule. Thus, M cannot sit in the row 1 aisle seat, which eliminates answer choice (E).
The remaining two answer choices—answer choices (B) and (C)—address row 2, of which the window seat is the only remaining seat available. If K sits in the row 2 window seat, then from the fourth rule M would have to sit in the row 3 window seat, forcing I to sit in row 1. But, this causes a violation of the condition in the question stem, and thus K cannot sit in row 2. This information eliminates answer choice (B), and thus answer choice (C) is correct.
The following hypothetical shows that answer choice (C) is possible:
You do not have the required permissions to view the files attached to this post.
Dave Killoran
PowerScore Test Preparation
Follow me on X/Twitter at http://twitter.com/DaveKilloran
My LSAT Articles: http://blog.powerscore.com/lsat/author/dave-killoran
PowerScore Podcast: http://www.powerscore.com/lsat/podcast/
PowerScore Test Preparation
Follow me on X/Twitter at http://twitter.com/DaveKilloran
My LSAT Articles: http://blog.powerscore.com/lsat/author/dave-killoran
PowerScore Podcast: http://www.powerscore.com/lsat/podcast/